LSAT and Law School Admissions Forum

Get expert LSAT preparation and law school admissions advice from PowerScore Test Preparation.

 Administrator
PowerScore Staff
  • PowerScore Staff
  • Posts: 8917
  • Joined: Feb 02, 2011
|
#25640
Complete Question Explanation
(The complete setup for this game can be found here: lsat/viewtopic.php?t=7800)

The correct answer choice is (E)

If G is visited first and J – fifth, we must also ensure that G is visited fourth, in compliance with the fourth rule:
june07_game_3_#14_diagram_1.png
To ensure that the M — G — M sequence is satisfied in this solution, M must be visited sixth, and either second or third:
june07_game_3_#14_diagram_2.png
Considering this is a Must Be True question, you should immediately check to see if your inference involving M is reflected in any of the answer choices. It is so in answer choice (E), validating that answer choice as correct.

Answer choice (A): This answer choice is not necessarily true. While J could be visited second, it does not necessarily have to be visited second. This answer choice tries to exploit a Mistaken Reversal™ error students may make with the fourth rule.

Answer choice (B): This answer choice is not necessarily true, because M or J could be visited second as well.

Answer choice (C): This answer choice is not necessarily true. M could be visited in week 2.

Answer choice (D): This answer choice must be false, because M must be visited sixth.

Answer choice (E): This is the correct answer choice, as explained above.
You do not have the required permissions to view the files attached to this post.
 martinbeslu
  • Posts: 49
  • Joined: Aug 09, 2017
|
#43283
It looks to me like J, M, or T could be second. Why is it that J could not be second?

For example, GJMGJMT

I'm sure there is something I'm missing but this seems to fit all of the rules.
User avatar
 Jonathan Evans
PowerScore Staff
  • PowerScore Staff
  • Posts: 726
  • Joined: Jun 09, 2016
|
#43631
Great catch, MartinBeslu! You're absolutely right. I have corrected the explanation above to reflect the possibility that J could be visited 2nd. Thank you for sharing your observation. We appreciate it!

Get the most out of your LSAT Prep Plus subscription.

Analyze and track your performance with our Testing and Analytics Package.